Multiplying summation with same indices and limits












1














What would be
$(1-sum limits_{k=0}^m x^k )(1-sum limits_{k=0}^m y^k ) ?$



I dont understand how can I multiply summation of same indices. I checked "multiplication of finite sum (inner product space)" this post but it is different than my case.



Any suggestion?










share|cite|improve this question






















  • These are both just partial sums of a geometric series, and $$sum_{k = 0}^m x^k = frac{1 - x^{m + 1}}{1 - x}.$$
    – T. Bongers
    Nov 19 '18 at 0:23










  • k is simply a dummy. For either sum any letter could have been used. The k's in the two sums have no relation to each other.
    – herb steinberg
    Nov 19 '18 at 0:42










  • As per my case, both summation have same k and same limits. I am not sure how to evaluate $sum limits_{k=0}^{m}sum limits_{k=0}^{m}x^ky^k$
    – hakkunamattata
    Nov 19 '18 at 0:54










  • @hakkunamattata As herb states above, it is not the "same" $k$. The summation indices are dummy variables, $sum_{k=0}^m x^k = sum_{j=0}^m x^k=sum_{heartsuit=0}^m x^{heartsuit}.$So $$left(sum_{k=0}^m x^kright)left(sum_{k=0}^m y^kright)=left(sum_{k=0}^m x^kright)left(sum_{ell=0}^m y^ellright)=sum_{k=0}^m sum_{ell=0}^m x^ky^ell$$
    – Clement C.
    Nov 19 '18 at 1:29












  • I suggest you work this out by hand for $m=2$ or $3$. Often summations with $Sigma$ are clearer if you do a small special case. For infinite sums write $a_1 + a_2 + cdots$.
    – Ethan Bolker
    Nov 19 '18 at 1:57
















1














What would be
$(1-sum limits_{k=0}^m x^k )(1-sum limits_{k=0}^m y^k ) ?$



I dont understand how can I multiply summation of same indices. I checked "multiplication of finite sum (inner product space)" this post but it is different than my case.



Any suggestion?










share|cite|improve this question






















  • These are both just partial sums of a geometric series, and $$sum_{k = 0}^m x^k = frac{1 - x^{m + 1}}{1 - x}.$$
    – T. Bongers
    Nov 19 '18 at 0:23










  • k is simply a dummy. For either sum any letter could have been used. The k's in the two sums have no relation to each other.
    – herb steinberg
    Nov 19 '18 at 0:42










  • As per my case, both summation have same k and same limits. I am not sure how to evaluate $sum limits_{k=0}^{m}sum limits_{k=0}^{m}x^ky^k$
    – hakkunamattata
    Nov 19 '18 at 0:54










  • @hakkunamattata As herb states above, it is not the "same" $k$. The summation indices are dummy variables, $sum_{k=0}^m x^k = sum_{j=0}^m x^k=sum_{heartsuit=0}^m x^{heartsuit}.$So $$left(sum_{k=0}^m x^kright)left(sum_{k=0}^m y^kright)=left(sum_{k=0}^m x^kright)left(sum_{ell=0}^m y^ellright)=sum_{k=0}^m sum_{ell=0}^m x^ky^ell$$
    – Clement C.
    Nov 19 '18 at 1:29












  • I suggest you work this out by hand for $m=2$ or $3$. Often summations with $Sigma$ are clearer if you do a small special case. For infinite sums write $a_1 + a_2 + cdots$.
    – Ethan Bolker
    Nov 19 '18 at 1:57














1












1








1


0





What would be
$(1-sum limits_{k=0}^m x^k )(1-sum limits_{k=0}^m y^k ) ?$



I dont understand how can I multiply summation of same indices. I checked "multiplication of finite sum (inner product space)" this post but it is different than my case.



Any suggestion?










share|cite|improve this question













What would be
$(1-sum limits_{k=0}^m x^k )(1-sum limits_{k=0}^m y^k ) ?$



I dont understand how can I multiply summation of same indices. I checked "multiplication of finite sum (inner product space)" this post but it is different than my case.



Any suggestion?







summation summation-method






share|cite|improve this question













share|cite|improve this question











share|cite|improve this question




share|cite|improve this question










asked Nov 19 '18 at 0:15









hakkunamattata

475




475












  • These are both just partial sums of a geometric series, and $$sum_{k = 0}^m x^k = frac{1 - x^{m + 1}}{1 - x}.$$
    – T. Bongers
    Nov 19 '18 at 0:23










  • k is simply a dummy. For either sum any letter could have been used. The k's in the two sums have no relation to each other.
    – herb steinberg
    Nov 19 '18 at 0:42










  • As per my case, both summation have same k and same limits. I am not sure how to evaluate $sum limits_{k=0}^{m}sum limits_{k=0}^{m}x^ky^k$
    – hakkunamattata
    Nov 19 '18 at 0:54










  • @hakkunamattata As herb states above, it is not the "same" $k$. The summation indices are dummy variables, $sum_{k=0}^m x^k = sum_{j=0}^m x^k=sum_{heartsuit=0}^m x^{heartsuit}.$So $$left(sum_{k=0}^m x^kright)left(sum_{k=0}^m y^kright)=left(sum_{k=0}^m x^kright)left(sum_{ell=0}^m y^ellright)=sum_{k=0}^m sum_{ell=0}^m x^ky^ell$$
    – Clement C.
    Nov 19 '18 at 1:29












  • I suggest you work this out by hand for $m=2$ or $3$. Often summations with $Sigma$ are clearer if you do a small special case. For infinite sums write $a_1 + a_2 + cdots$.
    – Ethan Bolker
    Nov 19 '18 at 1:57


















  • These are both just partial sums of a geometric series, and $$sum_{k = 0}^m x^k = frac{1 - x^{m + 1}}{1 - x}.$$
    – T. Bongers
    Nov 19 '18 at 0:23










  • k is simply a dummy. For either sum any letter could have been used. The k's in the two sums have no relation to each other.
    – herb steinberg
    Nov 19 '18 at 0:42










  • As per my case, both summation have same k and same limits. I am not sure how to evaluate $sum limits_{k=0}^{m}sum limits_{k=0}^{m}x^ky^k$
    – hakkunamattata
    Nov 19 '18 at 0:54










  • @hakkunamattata As herb states above, it is not the "same" $k$. The summation indices are dummy variables, $sum_{k=0}^m x^k = sum_{j=0}^m x^k=sum_{heartsuit=0}^m x^{heartsuit}.$So $$left(sum_{k=0}^m x^kright)left(sum_{k=0}^m y^kright)=left(sum_{k=0}^m x^kright)left(sum_{ell=0}^m y^ellright)=sum_{k=0}^m sum_{ell=0}^m x^ky^ell$$
    – Clement C.
    Nov 19 '18 at 1:29












  • I suggest you work this out by hand for $m=2$ or $3$. Often summations with $Sigma$ are clearer if you do a small special case. For infinite sums write $a_1 + a_2 + cdots$.
    – Ethan Bolker
    Nov 19 '18 at 1:57
















These are both just partial sums of a geometric series, and $$sum_{k = 0}^m x^k = frac{1 - x^{m + 1}}{1 - x}.$$
– T. Bongers
Nov 19 '18 at 0:23




These are both just partial sums of a geometric series, and $$sum_{k = 0}^m x^k = frac{1 - x^{m + 1}}{1 - x}.$$
– T. Bongers
Nov 19 '18 at 0:23












k is simply a dummy. For either sum any letter could have been used. The k's in the two sums have no relation to each other.
– herb steinberg
Nov 19 '18 at 0:42




k is simply a dummy. For either sum any letter could have been used. The k's in the two sums have no relation to each other.
– herb steinberg
Nov 19 '18 at 0:42












As per my case, both summation have same k and same limits. I am not sure how to evaluate $sum limits_{k=0}^{m}sum limits_{k=0}^{m}x^ky^k$
– hakkunamattata
Nov 19 '18 at 0:54




As per my case, both summation have same k and same limits. I am not sure how to evaluate $sum limits_{k=0}^{m}sum limits_{k=0}^{m}x^ky^k$
– hakkunamattata
Nov 19 '18 at 0:54












@hakkunamattata As herb states above, it is not the "same" $k$. The summation indices are dummy variables, $sum_{k=0}^m x^k = sum_{j=0}^m x^k=sum_{heartsuit=0}^m x^{heartsuit}.$So $$left(sum_{k=0}^m x^kright)left(sum_{k=0}^m y^kright)=left(sum_{k=0}^m x^kright)left(sum_{ell=0}^m y^ellright)=sum_{k=0}^m sum_{ell=0}^m x^ky^ell$$
– Clement C.
Nov 19 '18 at 1:29






@hakkunamattata As herb states above, it is not the "same" $k$. The summation indices are dummy variables, $sum_{k=0}^m x^k = sum_{j=0}^m x^k=sum_{heartsuit=0}^m x^{heartsuit}.$So $$left(sum_{k=0}^m x^kright)left(sum_{k=0}^m y^kright)=left(sum_{k=0}^m x^kright)left(sum_{ell=0}^m y^ellright)=sum_{k=0}^m sum_{ell=0}^m x^ky^ell$$
– Clement C.
Nov 19 '18 at 1:29














I suggest you work this out by hand for $m=2$ or $3$. Often summations with $Sigma$ are clearer if you do a small special case. For infinite sums write $a_1 + a_2 + cdots$.
– Ethan Bolker
Nov 19 '18 at 1:57




I suggest you work this out by hand for $m=2$ or $3$. Often summations with $Sigma$ are clearer if you do a small special case. For infinite sums write $a_1 + a_2 + cdots$.
– Ethan Bolker
Nov 19 '18 at 1:57










2 Answers
2






active

oldest

votes


















0














Let's examine series multiplication first:
$$A=sum_{igeq0}a_i=a_0+a_1+a_2+dots$$
$$B=sum_{igeq0}b_i=b_0+b_1+b_2+dots$$
$$AB=big(a_0+a_1+dotsbig)B$$
$$AB=a_0S_2+big(a_1+a_2+dotsbig)B$$
$$AB=a_0S_2+a_1S_2+big(a_2+a_3+dotsbig)B$$
$$AB=a_0S_2+a_1S_2+a_2S_2+big(a_3+a_4+dotsbig)B$$
This pattern continues:
$$AB=a_0B+a_1B+a_2B+a_3B+dots$$
$$AB=sum_{igeq0}a_iB$$
Now note the following:
$$a_iB=a_isum_{kgeq0}b_k$$
$$a_iB=a_ibig(b_0+b_1+b_2+dotsbig)$$
$$a_iB=a_ib_0+a_ib_1+a_ib_2+dots$$
$$a_iB=sum_{kgeq0}a_ib_k$$
Plugging in:
$$AB=sum_{igeq0}sum_{kgeq0}a_ib_k$$
Since $i$ is independent of $k$, and they belong to the same set (namely ${xinBbb Z:xgeq0})$, we know that
$$
begin{align}
AB & = a_0b_0+a_0b_1+a_0b_2+dots \
& + a_1b_0+a_1b_1+a_1b_2+dots \
& + a_2b_0+a_2b_1+a_2b_2+dots \
& +dots
end{align}
$$

Which can be greatly abbreviated:
$$AB=sum_{i,kin S}a_ib_k$$
Where
$$S={xinBbb Z:xgeq0}={0,1,2,dots}$$





Now we can move onto something more related to your problem:
$$A=sum_{i=0}^{m}a_i$$
$$B=sum_{i=0}^{m}b_i$$
These are just like the case above:
$$AB=sum_{i,kin S}a_ib_k$$
Where $$S={xinBbb Z:0leq xleq m}$$
And of course the fact
$$(1-A)(1-B)=AB-A-B+1$$
still holds when $A$ and $B$ are series. But one should note:
$$-A=-a_0-a_1-a_2-dots$$
and not $$-A=-a_0+a_1+a_2+dots$$






share|cite|improve this answer





























    0














    The index variables $k$ are so-called bound variables. This means that their scope (i.e. range of validity) is determined by their sigma-operator $sum$ and the operator precedence rules.




    The following representations are valid
    begin{align*}
    left(1+sum_{k=0}^mx^kright)left(1+sum_{k=0}^my^kright)&=
    left(1+color{green}{left(sum_{k=0}^mx^kright)}right)left(1+color{blue}{left(sum_{k=0}^my^kright)}right)tag{1}\
    &=left(1+sum_{k=0}^mx^kright)left(1+sum_{color{blue}{j=0}}^my^{color{blue}{j}}right)tag{2}
    end{align*}




    Comment:




    • In (1) we present the scope of each of the index variables somewhat more clearly by using inner parenthesis and the colors green and blue.


    • In (2) we denote the index variable of the right-most sum with $j$.




    Hint: It is often convenient to give different index variables different names, even if they have no overlapping scope. This usually enhances readability.







    share|cite|improve this answer























      Your Answer





      StackExchange.ifUsing("editor", function () {
      return StackExchange.using("mathjaxEditing", function () {
      StackExchange.MarkdownEditor.creationCallbacks.add(function (editor, postfix) {
      StackExchange.mathjaxEditing.prepareWmdForMathJax(editor, postfix, [["$", "$"], ["\\(","\\)"]]);
      });
      });
      }, "mathjax-editing");

      StackExchange.ready(function() {
      var channelOptions = {
      tags: "".split(" "),
      id: "69"
      };
      initTagRenderer("".split(" "), "".split(" "), channelOptions);

      StackExchange.using("externalEditor", function() {
      // Have to fire editor after snippets, if snippets enabled
      if (StackExchange.settings.snippets.snippetsEnabled) {
      StackExchange.using("snippets", function() {
      createEditor();
      });
      }
      else {
      createEditor();
      }
      });

      function createEditor() {
      StackExchange.prepareEditor({
      heartbeatType: 'answer',
      autoActivateHeartbeat: false,
      convertImagesToLinks: true,
      noModals: true,
      showLowRepImageUploadWarning: true,
      reputationToPostImages: 10,
      bindNavPrevention: true,
      postfix: "",
      imageUploader: {
      brandingHtml: "Powered by u003ca class="icon-imgur-white" href="https://imgur.com/"u003eu003c/au003e",
      contentPolicyHtml: "User contributions licensed under u003ca href="https://creativecommons.org/licenses/by-sa/3.0/"u003ecc by-sa 3.0 with attribution requiredu003c/au003e u003ca href="https://stackoverflow.com/legal/content-policy"u003e(content policy)u003c/au003e",
      allowUrls: true
      },
      noCode: true, onDemand: true,
      discardSelector: ".discard-answer"
      ,immediatelyShowMarkdownHelp:true
      });


      }
      });














      draft saved

      draft discarded


















      StackExchange.ready(
      function () {
      StackExchange.openid.initPostLogin('.new-post-login', 'https%3a%2f%2fmath.stackexchange.com%2fquestions%2f3004321%2fmultiplying-summation-with-same-indices-and-limits%23new-answer', 'question_page');
      }
      );

      Post as a guest















      Required, but never shown

























      2 Answers
      2






      active

      oldest

      votes








      2 Answers
      2






      active

      oldest

      votes









      active

      oldest

      votes






      active

      oldest

      votes









      0














      Let's examine series multiplication first:
      $$A=sum_{igeq0}a_i=a_0+a_1+a_2+dots$$
      $$B=sum_{igeq0}b_i=b_0+b_1+b_2+dots$$
      $$AB=big(a_0+a_1+dotsbig)B$$
      $$AB=a_0S_2+big(a_1+a_2+dotsbig)B$$
      $$AB=a_0S_2+a_1S_2+big(a_2+a_3+dotsbig)B$$
      $$AB=a_0S_2+a_1S_2+a_2S_2+big(a_3+a_4+dotsbig)B$$
      This pattern continues:
      $$AB=a_0B+a_1B+a_2B+a_3B+dots$$
      $$AB=sum_{igeq0}a_iB$$
      Now note the following:
      $$a_iB=a_isum_{kgeq0}b_k$$
      $$a_iB=a_ibig(b_0+b_1+b_2+dotsbig)$$
      $$a_iB=a_ib_0+a_ib_1+a_ib_2+dots$$
      $$a_iB=sum_{kgeq0}a_ib_k$$
      Plugging in:
      $$AB=sum_{igeq0}sum_{kgeq0}a_ib_k$$
      Since $i$ is independent of $k$, and they belong to the same set (namely ${xinBbb Z:xgeq0})$, we know that
      $$
      begin{align}
      AB & = a_0b_0+a_0b_1+a_0b_2+dots \
      & + a_1b_0+a_1b_1+a_1b_2+dots \
      & + a_2b_0+a_2b_1+a_2b_2+dots \
      & +dots
      end{align}
      $$

      Which can be greatly abbreviated:
      $$AB=sum_{i,kin S}a_ib_k$$
      Where
      $$S={xinBbb Z:xgeq0}={0,1,2,dots}$$





      Now we can move onto something more related to your problem:
      $$A=sum_{i=0}^{m}a_i$$
      $$B=sum_{i=0}^{m}b_i$$
      These are just like the case above:
      $$AB=sum_{i,kin S}a_ib_k$$
      Where $$S={xinBbb Z:0leq xleq m}$$
      And of course the fact
      $$(1-A)(1-B)=AB-A-B+1$$
      still holds when $A$ and $B$ are series. But one should note:
      $$-A=-a_0-a_1-a_2-dots$$
      and not $$-A=-a_0+a_1+a_2+dots$$






      share|cite|improve this answer


























        0














        Let's examine series multiplication first:
        $$A=sum_{igeq0}a_i=a_0+a_1+a_2+dots$$
        $$B=sum_{igeq0}b_i=b_0+b_1+b_2+dots$$
        $$AB=big(a_0+a_1+dotsbig)B$$
        $$AB=a_0S_2+big(a_1+a_2+dotsbig)B$$
        $$AB=a_0S_2+a_1S_2+big(a_2+a_3+dotsbig)B$$
        $$AB=a_0S_2+a_1S_2+a_2S_2+big(a_3+a_4+dotsbig)B$$
        This pattern continues:
        $$AB=a_0B+a_1B+a_2B+a_3B+dots$$
        $$AB=sum_{igeq0}a_iB$$
        Now note the following:
        $$a_iB=a_isum_{kgeq0}b_k$$
        $$a_iB=a_ibig(b_0+b_1+b_2+dotsbig)$$
        $$a_iB=a_ib_0+a_ib_1+a_ib_2+dots$$
        $$a_iB=sum_{kgeq0}a_ib_k$$
        Plugging in:
        $$AB=sum_{igeq0}sum_{kgeq0}a_ib_k$$
        Since $i$ is independent of $k$, and they belong to the same set (namely ${xinBbb Z:xgeq0})$, we know that
        $$
        begin{align}
        AB & = a_0b_0+a_0b_1+a_0b_2+dots \
        & + a_1b_0+a_1b_1+a_1b_2+dots \
        & + a_2b_0+a_2b_1+a_2b_2+dots \
        & +dots
        end{align}
        $$

        Which can be greatly abbreviated:
        $$AB=sum_{i,kin S}a_ib_k$$
        Where
        $$S={xinBbb Z:xgeq0}={0,1,2,dots}$$





        Now we can move onto something more related to your problem:
        $$A=sum_{i=0}^{m}a_i$$
        $$B=sum_{i=0}^{m}b_i$$
        These are just like the case above:
        $$AB=sum_{i,kin S}a_ib_k$$
        Where $$S={xinBbb Z:0leq xleq m}$$
        And of course the fact
        $$(1-A)(1-B)=AB-A-B+1$$
        still holds when $A$ and $B$ are series. But one should note:
        $$-A=-a_0-a_1-a_2-dots$$
        and not $$-A=-a_0+a_1+a_2+dots$$






        share|cite|improve this answer
























          0












          0








          0






          Let's examine series multiplication first:
          $$A=sum_{igeq0}a_i=a_0+a_1+a_2+dots$$
          $$B=sum_{igeq0}b_i=b_0+b_1+b_2+dots$$
          $$AB=big(a_0+a_1+dotsbig)B$$
          $$AB=a_0S_2+big(a_1+a_2+dotsbig)B$$
          $$AB=a_0S_2+a_1S_2+big(a_2+a_3+dotsbig)B$$
          $$AB=a_0S_2+a_1S_2+a_2S_2+big(a_3+a_4+dotsbig)B$$
          This pattern continues:
          $$AB=a_0B+a_1B+a_2B+a_3B+dots$$
          $$AB=sum_{igeq0}a_iB$$
          Now note the following:
          $$a_iB=a_isum_{kgeq0}b_k$$
          $$a_iB=a_ibig(b_0+b_1+b_2+dotsbig)$$
          $$a_iB=a_ib_0+a_ib_1+a_ib_2+dots$$
          $$a_iB=sum_{kgeq0}a_ib_k$$
          Plugging in:
          $$AB=sum_{igeq0}sum_{kgeq0}a_ib_k$$
          Since $i$ is independent of $k$, and they belong to the same set (namely ${xinBbb Z:xgeq0})$, we know that
          $$
          begin{align}
          AB & = a_0b_0+a_0b_1+a_0b_2+dots \
          & + a_1b_0+a_1b_1+a_1b_2+dots \
          & + a_2b_0+a_2b_1+a_2b_2+dots \
          & +dots
          end{align}
          $$

          Which can be greatly abbreviated:
          $$AB=sum_{i,kin S}a_ib_k$$
          Where
          $$S={xinBbb Z:xgeq0}={0,1,2,dots}$$





          Now we can move onto something more related to your problem:
          $$A=sum_{i=0}^{m}a_i$$
          $$B=sum_{i=0}^{m}b_i$$
          These are just like the case above:
          $$AB=sum_{i,kin S}a_ib_k$$
          Where $$S={xinBbb Z:0leq xleq m}$$
          And of course the fact
          $$(1-A)(1-B)=AB-A-B+1$$
          still holds when $A$ and $B$ are series. But one should note:
          $$-A=-a_0-a_1-a_2-dots$$
          and not $$-A=-a_0+a_1+a_2+dots$$






          share|cite|improve this answer












          Let's examine series multiplication first:
          $$A=sum_{igeq0}a_i=a_0+a_1+a_2+dots$$
          $$B=sum_{igeq0}b_i=b_0+b_1+b_2+dots$$
          $$AB=big(a_0+a_1+dotsbig)B$$
          $$AB=a_0S_2+big(a_1+a_2+dotsbig)B$$
          $$AB=a_0S_2+a_1S_2+big(a_2+a_3+dotsbig)B$$
          $$AB=a_0S_2+a_1S_2+a_2S_2+big(a_3+a_4+dotsbig)B$$
          This pattern continues:
          $$AB=a_0B+a_1B+a_2B+a_3B+dots$$
          $$AB=sum_{igeq0}a_iB$$
          Now note the following:
          $$a_iB=a_isum_{kgeq0}b_k$$
          $$a_iB=a_ibig(b_0+b_1+b_2+dotsbig)$$
          $$a_iB=a_ib_0+a_ib_1+a_ib_2+dots$$
          $$a_iB=sum_{kgeq0}a_ib_k$$
          Plugging in:
          $$AB=sum_{igeq0}sum_{kgeq0}a_ib_k$$
          Since $i$ is independent of $k$, and they belong to the same set (namely ${xinBbb Z:xgeq0})$, we know that
          $$
          begin{align}
          AB & = a_0b_0+a_0b_1+a_0b_2+dots \
          & + a_1b_0+a_1b_1+a_1b_2+dots \
          & + a_2b_0+a_2b_1+a_2b_2+dots \
          & +dots
          end{align}
          $$

          Which can be greatly abbreviated:
          $$AB=sum_{i,kin S}a_ib_k$$
          Where
          $$S={xinBbb Z:xgeq0}={0,1,2,dots}$$





          Now we can move onto something more related to your problem:
          $$A=sum_{i=0}^{m}a_i$$
          $$B=sum_{i=0}^{m}b_i$$
          These are just like the case above:
          $$AB=sum_{i,kin S}a_ib_k$$
          Where $$S={xinBbb Z:0leq xleq m}$$
          And of course the fact
          $$(1-A)(1-B)=AB-A-B+1$$
          still holds when $A$ and $B$ are series. But one should note:
          $$-A=-a_0-a_1-a_2-dots$$
          and not $$-A=-a_0+a_1+a_2+dots$$







          share|cite|improve this answer












          share|cite|improve this answer



          share|cite|improve this answer










          answered Nov 19 '18 at 1:52









          clathratus

          3,126331




          3,126331























              0














              The index variables $k$ are so-called bound variables. This means that their scope (i.e. range of validity) is determined by their sigma-operator $sum$ and the operator precedence rules.




              The following representations are valid
              begin{align*}
              left(1+sum_{k=0}^mx^kright)left(1+sum_{k=0}^my^kright)&=
              left(1+color{green}{left(sum_{k=0}^mx^kright)}right)left(1+color{blue}{left(sum_{k=0}^my^kright)}right)tag{1}\
              &=left(1+sum_{k=0}^mx^kright)left(1+sum_{color{blue}{j=0}}^my^{color{blue}{j}}right)tag{2}
              end{align*}




              Comment:




              • In (1) we present the scope of each of the index variables somewhat more clearly by using inner parenthesis and the colors green and blue.


              • In (2) we denote the index variable of the right-most sum with $j$.




              Hint: It is often convenient to give different index variables different names, even if they have no overlapping scope. This usually enhances readability.







              share|cite|improve this answer




























                0














                The index variables $k$ are so-called bound variables. This means that their scope (i.e. range of validity) is determined by their sigma-operator $sum$ and the operator precedence rules.




                The following representations are valid
                begin{align*}
                left(1+sum_{k=0}^mx^kright)left(1+sum_{k=0}^my^kright)&=
                left(1+color{green}{left(sum_{k=0}^mx^kright)}right)left(1+color{blue}{left(sum_{k=0}^my^kright)}right)tag{1}\
                &=left(1+sum_{k=0}^mx^kright)left(1+sum_{color{blue}{j=0}}^my^{color{blue}{j}}right)tag{2}
                end{align*}




                Comment:




                • In (1) we present the scope of each of the index variables somewhat more clearly by using inner parenthesis and the colors green and blue.


                • In (2) we denote the index variable of the right-most sum with $j$.




                Hint: It is often convenient to give different index variables different names, even if they have no overlapping scope. This usually enhances readability.







                share|cite|improve this answer


























                  0












                  0








                  0






                  The index variables $k$ are so-called bound variables. This means that their scope (i.e. range of validity) is determined by their sigma-operator $sum$ and the operator precedence rules.




                  The following representations are valid
                  begin{align*}
                  left(1+sum_{k=0}^mx^kright)left(1+sum_{k=0}^my^kright)&=
                  left(1+color{green}{left(sum_{k=0}^mx^kright)}right)left(1+color{blue}{left(sum_{k=0}^my^kright)}right)tag{1}\
                  &=left(1+sum_{k=0}^mx^kright)left(1+sum_{color{blue}{j=0}}^my^{color{blue}{j}}right)tag{2}
                  end{align*}




                  Comment:




                  • In (1) we present the scope of each of the index variables somewhat more clearly by using inner parenthesis and the colors green and blue.


                  • In (2) we denote the index variable of the right-most sum with $j$.




                  Hint: It is often convenient to give different index variables different names, even if they have no overlapping scope. This usually enhances readability.







                  share|cite|improve this answer














                  The index variables $k$ are so-called bound variables. This means that their scope (i.e. range of validity) is determined by their sigma-operator $sum$ and the operator precedence rules.




                  The following representations are valid
                  begin{align*}
                  left(1+sum_{k=0}^mx^kright)left(1+sum_{k=0}^my^kright)&=
                  left(1+color{green}{left(sum_{k=0}^mx^kright)}right)left(1+color{blue}{left(sum_{k=0}^my^kright)}right)tag{1}\
                  &=left(1+sum_{k=0}^mx^kright)left(1+sum_{color{blue}{j=0}}^my^{color{blue}{j}}right)tag{2}
                  end{align*}




                  Comment:




                  • In (1) we present the scope of each of the index variables somewhat more clearly by using inner parenthesis and the colors green and blue.


                  • In (2) we denote the index variable of the right-most sum with $j$.




                  Hint: It is often convenient to give different index variables different names, even if they have no overlapping scope. This usually enhances readability.








                  share|cite|improve this answer














                  share|cite|improve this answer



                  share|cite|improve this answer








                  edited Nov 19 '18 at 16:38

























                  answered Nov 19 '18 at 12:52









                  Markus Scheuer

                  60k455143




                  60k455143






























                      draft saved

                      draft discarded




















































                      Thanks for contributing an answer to Mathematics Stack Exchange!


                      • Please be sure to answer the question. Provide details and share your research!

                      But avoid



                      • Asking for help, clarification, or responding to other answers.

                      • Making statements based on opinion; back them up with references or personal experience.


                      Use MathJax to format equations. MathJax reference.


                      To learn more, see our tips on writing great answers.





                      Some of your past answers have not been well-received, and you're in danger of being blocked from answering.


                      Please pay close attention to the following guidance:


                      • Please be sure to answer the question. Provide details and share your research!

                      But avoid



                      • Asking for help, clarification, or responding to other answers.

                      • Making statements based on opinion; back them up with references or personal experience.


                      To learn more, see our tips on writing great answers.




                      draft saved


                      draft discarded














                      StackExchange.ready(
                      function () {
                      StackExchange.openid.initPostLogin('.new-post-login', 'https%3a%2f%2fmath.stackexchange.com%2fquestions%2f3004321%2fmultiplying-summation-with-same-indices-and-limits%23new-answer', 'question_page');
                      }
                      );

                      Post as a guest















                      Required, but never shown





















































                      Required, but never shown














                      Required, but never shown












                      Required, but never shown







                      Required, but never shown

































                      Required, but never shown














                      Required, but never shown












                      Required, but never shown







                      Required, but never shown







                      Popular posts from this blog

                      QoS: MAC-Priority for clients behind a repeater

                      Ивакино (Тотемский район)

                      Can't locate Autom4te/ChannelDefs.pm in @INC (when it definitely is there)